This is a pseudo-sufficient assumption question.
P:All leaders in major parities oppose the bill
C:The bill will most likely fail to pass.
Target answer to connect no support with fail to pass.
I found this question especially tricky because of what I perceive to be a logic gap in the stimulus. The critics argument can be boiled down to (Justified public funding) -& ...
http://7sage.com/lsat_explanations/lsat-30-section-2-question-20/
Hello guys, this MBT question actually comes from the MBT course video. Most people have the question about the last sentence "even its critics acknowledge." I have read all the ...
I'm super struggling with this question simply because I feel like the answer makes a big jump. It almost seem like its a sufficient assumption question. I just don't see where the stimulus indicates where the public support in ...
The question stem for this question asks us to pick an answer choice that shows that the explanation we were given in the stimulus is only a "partial one." I was doing this question as part of my weakening problem set. I read through the stimulus and was ...